[Physics] Time ordering operator and derivative with respect to time

operatorsquantum mechanicsquantum-field-theorytime evolutionunitarity

In the book Quantum field theory and the standard model from Schwartz, it is written on page 87 some results using time ordering operator.

We have the following operator:

$$ U(t,t_0)=T \exp\biggl(-\mathrm i \int_{t_0}^t V_I(u)\, \mathrm{d}u\biggr)$$

It is said the following things:

7.2.2 $U$ relations

It is convenient to abbreviate $U$ with
$$U_{21} \equiv U(t_2, t_1) = T\biggl\{\exp\biggl[-t\int_{t_1}^{t_2} \mathrm{d}t'\,V_I(t')\biggr]\biggr\}.\tag{7.46}$$
Remember that in field theory we always have later times on the left. It follows that
$$\begin{align}
U_{21}U_{12} &= 1, \tag{7.47} \\
U_{21}^{-1} = U_{21}^{\dagger} &= U_{12} \tag{7.48}
\end{align}$$
and for $t_1 < t_2 < t_3$
$$U_{32} U_{21} = U_{31}. \tag{7.49}$$
Multiplying this by $U_{12}$ on the right, we find
$$U_{31}U_{12} = U_{32}, \tag{7.50}$$

Ok, I don't understand their "proof" of (7.47) and (7.49).

Remember that in field theory we always have later times on the left. It follows that:

Is it really a proof of the equations below? I don't get it.

Also, to prove that I would write the exponential in series and reason order by order but is there a better way to prove it? Because it is not really immediate (I don't know if it is possible to find a nice way to prove it).

Best Answer

The time-evolution operator in the interaction picture can be written as: $$U(t,t_{0})=e^{iH_{0}t}e^{-iH(t-t_{0})}e^{-iH_{0}t_{0}}$$ Using this we can write: $$U(t_{1},t_{2})U(t_{2},t_{0})=e^{iH_{0}t_{1}}e^{-iH(t_{1}-t_{2})}e^{-iH_{0}t_{2}}e^{iH_{0}t_{2}}e^{-iH(t_{2}-t_{0})}e^{-iH_{0}t_{0}}=e^{iH_{0}t_{1}}e^{-iH(t_{1}-t_{0})}e^{-iH_{0}t_{0}}$$ So: $$U(t_{1},t_{2})U(t_{2},t_{0})=U(t_{1},t_{0})$$

From Tomonaga-Schwinger equation: $$i\partial_{t}U(t,t_{0})=H_{I}(t)U(t,t_{0})$$ We can write the time-evolution operator using the initial condition $U(t_{0},t_{0})=1$: $$U(t,t_{0})=1-i\int_{t_{0}}^{t}dt_{1}H_{I}(t_{1})U(t_{1},t_{0})$$ By iteration, we obtain that: $$U(t,t_{0})=1+(-i)\int_{t_{0}}^{t}dt_{1}H_{I}(t_{1})+(-i)^2\int_{t_{0}}^{t}dt_{1}\int_{t_{0}}^{t_{1}}dt_{2}H_{I}(t_{1})H_{I}(t_{2})+(-i)^3\int_{t_{0}}^{t}dt_{1}\int_{t_{0}}^{t_{1}}dt_{2}\int_{t_{0}}^{t_{2}}dt_{3}H_{I}(t_{1})H_{I}(t_{2})H_{I}(t_{3})+\dots$$ i.e., $$U(t,t_{0})=\sum_{i=0}^{\infty}(-i)^n\int_{t_{0}}^{t}dt_{1}\int_{t_{0}}^{t_{1}}dt_{2}\dots \int_{t_{0}}^{t_{n-1}}dt_{n}H_{I}(t_{1})H_{I}(t_{2})\dots H_{I}(t_{n})$$

$$U(t,t_{0})=\sum_{i=0}^{\infty}\frac{(-i)^n}{n!}\int_{t_{0}}^{t}dt_{1}\int_{t_{0}}^{t_{1}}dt_{2}\dots \int_{t_{0}}^{t_{n-1}}dt_{n}\mathcal{T}\left(H_{I}(t_{1})H_{I}(t_{2})\dots H_{I}(t_{n})\right)$$ $$U(t,t_{0})=\mathcal{T}\exp\left(-i\int_{t_{0}}^{t}dt'H_{I}(t')\right)$$